Difference between revisions of "2020 AMC 12B Problems/Problem 16"

(Solution 4)
(Redirected page to 2020 AMC 10B Problems/Problem 18)
(Tag: New redirect)
 
Line 1: Line 1:
==Problem==
+
#REDIRECT [[2020 AMC 10B Problems/Problem 18]]
An urn contains one red ball and one blue ball. A box of extra red and blue balls lie nearby. George performs the following operation four times: he draws a ball from the urn at random and then takes a ball of the same color from the box and returns those two matching balls to the urn.
 
After the four iterations the urn contains six balls. What is the probability that the urn contains three balls of each color?
 
<math>\textbf{(A)}\ \frac{1}{6} \qquad\textbf{(B)}\ \frac{1}{5} \qquad\textbf{(C)}\ \frac{1}{4} \qquad\textbf{(D)}\ \frac{1}{3} \qquad\textbf{(E)}\ \frac{1}{2}</math>
 
 
 
==Solution==
 
Let the probability that the urn ends up with more red balls be denoted <math>P(R)</math>. Since this is equal to the probability there are more blue balls, the probability there are equal amounts is <math>1-2P(R)</math>. <math>P(R) =</math> the probability no more blues are chosen plus the probability only 1 more blue is chosen. The first case, <math>P(\text{no more blues}) = \frac{1}{2}\cdot\frac{2}{3}\cdot\frac{3}{4}\cdot\frac{4}{5}=\frac{1}{5}</math>.
 
 
 
The second case, <math>P(\text{1 more blue}) = 4\cdot\frac{1\cdot1\cdot2\cdot3}{2\cdot3\cdot4\cdot5} = \frac{1}{5}</math>. Thus, the answer is <math>1-2\left(\frac{1}{5}+\frac{1}{5}\right)=1-\frac{4}{5}=\boxed{\textbf{(B)}\ \frac{1}{5}}</math>.
 
 
 
~JHawk0224
 
 
 
==Solution 2==
 
By conditional probability after 4 rounds we have 5 cases: RRRBBB, RRRRBB, RRBBBB, RRRRRB and RBBBBB. Thus the probability is <math>\frac{1}{5}</math>. Put <math>\boxed{B}</math>.
 
 
 
~FANYUCHEN20020715
 
 
 
Edited by Kinglogic
 
 
 
==Solution 3==
 
Here X stands for R or B, and Y for the remaining color.
 
After 3 rounds one can either have a 4+1 configuration (XXXXY), or 3+2 configuration (XXXYY). The probability of getting to XXXYYY from XXXYY is  <math>\frac{2}{5}</math>. Observe that the probability of arriving to 4+1 configuration is  <cmath>\frac{2}{3} \cdot \frac{3}{4} = \frac{1}{2}</cmath> (<math>\frac{2}{3}</math> to get from XXY to XXXY, <math>\frac{3}{4}</math> to get from XXXY to XXXXY). Thus the probability of arriving to 3+2 configuration is also <math>\frac{1}{2}</math>, and the answer is <cmath>\frac{1}{2} \cdot \frac{2}{5} = \boxed{\textbf{(B)}\ \frac{1}{5}}. </cmath>
 
 
 
==Solution 4==
 
We can try to use dynamic programming to solve this problem. (Informatics Olympiad hahaha)
 
 
 
We let <math>dp[i][j]</math> be the probability that we end up with <math>i</math> red balls and <math>j</math> blue balls.
 
Notice that there are only two ways that we can end up with <math>i</math> red balls and <math>j</math> blue balls: one is by fetching a red ball from the urn when we have <math>i - 1</math> red balls and <math>j</math> blue balls and the other is by fetching a blue ball from the urn when we have <math>i</math> red balls and <math>j - 1</math> blue balls.
 
 
 
Then we have
 
<math>dp[i][j] = \frac{i - 1}{i - 1 + j} dp[i - 1][j] + \frac{j - 1}{i - 1 + j} dp[i][j - 1]</math>
 
 
 
Then we start can with <math>dp[1][1] = 1</math> and try to compute <math>dp[3][3]</math>.
 
 
 
<cmath>\begin{array}{|c || c | c | c | c | c |}
 
\hline
 
i \text{\ \textbackslash\ } j & 1 & 2 & 3\\ \hline\hline
 
1 & 1 & 1/2 & 1/3\\ \hline
 
2 & 1/2 & 1/3 & 1/4\\ \hline
 
3 & 1/3 & 1/4 & 1/5\\ \hline
 
\end{array}</cmath>
 
The answer is <math>\boxed{\textbf{(B)}\ \frac{1}{5}}</math>.
 
 
 
(Solution by CircleOO)
 
 
 
==See Also==
 
 
 
{{AMC12 box|year=2020|ab=B|num-b=15|num-a=17}}
 
{{MAA Notice}}
 

Latest revision as of 21:17, 12 February 2020